GENERAL 4

Foro de discusion Sobre RFH

Moderador: Alberto

Avatar de Usuario
Curie
Rn
Rn
Mensajes: 1003
Registrado: 14 Dic 2008, 19:41

Mensaje por Curie »

Vaaaleeee!!!!!:salute: Mil graciassss .

Jo, odio la termoooooo :cry:
taconi23
N
N
Mensajes: 69
Registrado: 08 Ene 2009, 18:44

Mensaje por taconi23 »

una cosa mas que debes tener en cuenta curie es que lo primero que habria que hacer es calcular si la masa de agua tiene suficiente energia para fundir todo el hielo, yo ya supuse que si en funcion de la temperatura que debia alcanzar el equilibrio porque de lo contrario creo y de esto no estoy muy seguro la temperatura de equilibrio seria 0ºC pero esto lo lei un poco por encima espero que me explique y que responda a la pregunta que hiciste antes sobre si hay que suponer que todo el hielo se funde...
taconi23
N
N
Mensajes: 69
Registrado: 08 Ene 2009, 18:44

Mensaje por taconi23 »

Con respecto a la pregunta 35, no puedo asegurar que este bien pero te cuento mi teoria..
la probalibilidad de interaccion de rayos x es proporcional a \(Z^3\)
por lo tanto tendriamos
\(\frac{53^3}{7,4^3}\approx 367\)
por otro lado con respecto a la densidad la probabilidad es proporcional a la densidad si se dobla la densidad se dobla la probabilidad por lo tanto para el iodo es 4.93 veces mas probable por lo tanto la total es
\(P=367\cdot 4,93\approx 1.809\)

creo que es asi pero no toy seguro...
Avatar de Usuario
Curie
Rn
Rn
Mensajes: 1003
Registrado: 14 Dic 2008, 19:41

Mensaje por Curie »

Si tiene sentido...pero esq tal como lo ponen parace uno y pico, no mil y pico...Utiliza la coma como decimal... :?
taconi23
N
N
Mensajes: 69
Registrado: 08 Ene 2009, 18:44

Mensaje por taconi23 »

Para la 133 mi razonamiento es que cada 0.5ºC la potencia aumenta en 1mW. Partimos de 25ºC por lo tanto hasta 100ºC son 75, o sea 75/0.5=150mW otra forma no se me ocurre.
Avatar de Usuario
Curie
Rn
Rn
Mensajes: 1003
Registrado: 14 Dic 2008, 19:41

Mensaje por Curie »

Jue, vale!!!!No se me habia ocurrido,yo simplemente cogia la maxima... :?
Gracias!
taconi23
N
N
Mensajes: 69
Registrado: 08 Ene 2009, 18:44

Mensaje por taconi23 »

para la 89, a ver otra vez mi teoria .. tenemos un pendulo fisico por tanto el periodo de las oscilaciones responde a la expresion

\(T=2\pi\sqrt{\frac{I}{mgl}}\)

siendo \(l\) la distancia al centro de masas desde el extremo, es decir que si nuestro pendulo tiene una longitud L entonces \(L=l/2\)

I por tratarse de una varilla es \(I=\frac{1}{3}mL^2\)

si sustituimos los datos que tenemos de L=2 metros masa 1Kg tenemos

\(T=2\pi\sqrt{\frac{\frac{1}{3}mL^2}{mg\frac{L}{2}}}=2\pi\sqrt{\frac{\frac{1}{3}\cdot 1\cdot 2^2}{1\cdot 9,8\cdot \frac{2}{2}}}=2\pi\sqrt{\frac{4}{3\cdot 9,8}}\)

Sacando el 4 de la raiz

\(T=4\pi\sqrt{\frac{1}{3\cdot 9,8}}\)

ahora solo nos molesta el pi y el 9,8 pero \(\sqrt{9,8}=3,13\approx \pi\), por lo tanto pi se va y nos queda

\(\boxed{T=\frac{4}{\sqrt{3}}}\)

creo que es asi
taconi23
N
N
Mensajes: 69
Registrado: 08 Ene 2009, 18:44

Mensaje por taconi23 »

Avatar de Usuario
Ea
Rb
Rb
Mensajes: 366
Registrado: 25 Mar 2009, 21:00

Mensaje por Ea »

Hola, vamos a ver que nos queda por ahi que yo me sepa...

4. En esta la respuesta correcta que dan no son 130 años, son 88,1, y sale bien. Vamos a salir transtornaaaaaaaaaaaaaaaaaaos.

5. El coeficiente de dilatacion sera 0 en el punto donde la pendiente cambia de signo, que es en 4ºC, no? Quizá no he entendido tu razonamiento sobre esto, dimy, por qué dices que es 0 en 0ºC?

12. Curie, yo creo que esta es correcta, lo que se ha de conservar es el numero de nucleones total. Si no, qué sería de nuestra pobrecilla desintegración beta :cry: ....

19. La respuesta viene de considerar
\( V=\frac{Q+q}{4\pi\epsilon_0 R} \)
y despejar Q

35. Esta se hace como dice Taconi23, estoy deacuerdo, y da 1809, no 1.809. Ay, trastornaicos salimos de esta.

59. A mi si me da -v/2. Por otra parte, si antes de chocar la velocidad es v/2, y el choque es perfectamente elastico, despues de chocar la direccion de la velocidad no puede ser la misma.

98. Al meter la placa metalica el condensador se transforma en dos condensadores de longitud d/3 separados por un espacio de longitud d/3 donde no hay energía, pues no hay campo eléctrico. Entonces se calcula la capacidad equivalente de estos condensadores en serie, y con ésta, la energía, teniendo en cuenta que V no cambia. Da un 50% más.

123. Esta yo no la entiendo. La respuesta es 84.4 MeV cuando se considera 1 neutron, y no 2. Por qué?

191. Bueno, en realida lo que nos vendria muy bien que nos dieran en esta, seria el Z de cada uno. Asi sabríamos que ha de ser beta más por narices. Habria que aprenderse tooooooooooooooodoooos los Z de toooooooooodoooos los elementos. Excitante.


Si, sin duda, excitante, que gran domingo por la tarde... :pukeright:
Última edición por Ea el 26 Abr 2009, 19:55, editado 1 vez en total.
taconi23
N
N
Mensajes: 69
Registrado: 08 Ene 2009, 18:44

Mensaje por taconi23 »

Hola Ea en la 123 solo es un neutron porque la reaccion seria la siguiente

\(^{235}_{92}U+^1_0n\rightarrow ^{94}_{38}Sr+^{140}_{54}Xe+2\,^1_0n \)

por lo tanto uno de los neutrones se te cancella ya y solo te keda uno
Avatar de Usuario
Ea
Rb
Rb
Mensajes: 366
Registrado: 25 Mar 2009, 21:00

Mensaje por Ea »

Aaala claaaro, que es una fision. Eh gracias taconi23.
Dimy
Li
Mensajes: 25
Registrado: 06 Mar 2009, 17:40

Mensaje por Dimy »

creo que mi razonamiento sólo puede fallar si el coeficiente de dilatación sufre una discontinuidad al cambiar de fase el agua. me explico: el agua sólo tiene este coeficiente negativo entre 0 y 4ºC. al menos eso es lo que me estudié yo en su día. entonces, el agua a menos de 0ºC (vale, a presión atmosférica es hielo, pero ahí viene el posible fallo si es discontinuo en el cambio de fase). si alfa es continua, entonces por fuerza tiene que valer 0 a 0ºC (porque a más temperatura es negativo y a menos temperatura positivo).
espero que haya quedado más claro. de todas formas, la que es clarísimamente cierta es la respuesta 4, la de que un kilo de paja equilibra una balanza con un kilo de plomo. sólo intentaba ver alguna justificación para que dieran esa como buena. espero que sirva de algo. saludos!
Avatar de Usuario
Curie
Rn
Rn
Mensajes: 1003
Registrado: 14 Dic 2008, 19:41

Mensaje por Curie »

Bueno,muchas graciassssss.
Ya veo que tengo tendencia a apuntar las respuestas correstas mal...Como si no tuviera bastantes dudas con las que tengo mal, ahra me lio con las que tengo bien...jajajajaj.

Para la proxima prometo fijarme bien en lo que apuntoo, sorryyy... :cry:


Apasionante domingo, si.....
Avatar de Usuario
Curie
Rn
Rn
Mensajes: 1003
Registrado: 14 Dic 2008, 19:41

Mensaje por Curie »

Vale esque me he liado con la 4, a mi me sale 130,pero la correcta es 88,1.Ea como lo haces??
Avatar de Usuario
Ea
Rb
Rb
Mensajes: 366
Registrado: 25 Mar 2009, 21:00

Mensaje por Ea »

A ver, loca perdía (:wink:), se trata de la dilatacion del tiempo
\(t=\gamma t_p\),
siendo \(t=x/v=95.365.24.3600.3x19^8/2.2x10^8\) y \(\gamma=1.47\). Despejamos \(t_p\) y lo pasamos a años, y ya estamos.
Responder